Step 2 CK (CCSSA) Form 7

These answer explanations are and always will be free. However, given multiple email requests, I will post my Venmo (@Adam-Zakaria-SLO) if you want to send a few dollars to show your support for the website.

I also offer reasonably priced Study Guides and Personalized Study schedules, so please reach out using the Tutoring menu option or Study Guides and Personalized Study schedules menu option listed above if you would like personalized support.

Furthermore, I offer personal statement and application review services for residency applicants, so please reach out using the “Residency Advising and Application Preparation” menu option above if interested.

Lastly, please check out my Youtube channel (https://www.youtube.com/channel/UCT1Ukl4pm5QK9iw6h4MB_Hw/playlists) and the “Biostatistics Curriculum” option above for free videos and practice questions reviewing all the essential biostatistics topics covered on NBME exams. Good luck with your exams!

Exam section 1:

1: A previously healthy 9-year-old boy is brought to the physician…

Ankylosing spondylitis

  • Child who presents with inflammatory back pain (worse upon waking in morning) with involvement of sacroiliac joint and decreased flexion at waist (2 features that are highly associated/specific for ankylosing spondylitis) and elevated ESR (further supporting inflammatory back pain) with imaging showing vertebral body squaring and fusion of sacroiliac joint, most consistent with ankylosing spondylitis
  • Key idea: Back pain that is worse upon waking in morning = Inflammatory back pain = PAIR diagnoses: Psoriatic arthritis, Ankylosing spondylitis, Inflammatory bowel disease, Reiter/Reactive arthritis (at least on NBME exams)

2. A 15-year-old girl is brought to the physician because of low back…

Compression fracture

  • Young girl with a history of chronic corticosteroid use and cushingoid appearance who presents with constant back pain with vertebral point tenderness with imaging showing vertebrae with increased lucency, most consistent with a compression fracture in the setting of steroid-induced osteoporosis
  • Key idea: Vertebral compression fracture comes in 2 flavors (1) Chronic fracture: Painless with progressive kyphosis and loss of stature (2) Acute fracture: Low back pain with decreased spinal mobility + tenderness at affected level
  • Key idea: Causes of point tenderness over vertebral body includes compression vertebral fractures, osteomyelitis and metastatic disease to vertebral bone
  • Risk factors for osteoporosis –> compression fracture: Low weight (osteoporosis), increased age, women, chronic corticosteroid use or Cushing syndrome

3. Over the past month, a 72-year-old woman has had indigestion…

Exercise stress test

  • Patient with “indigestion” who has exertional chest pain that is unrelated to eating or relieved by antacids, racing concern for stable angina that should be worked up by exercise stress test

4. A previously healthy 62-year-old woman comes to the physician…

Barium swallow

  • Older woman with dysphagia to solids and liquids with intermittent regurgitation of undigested food and halitosis, most consistent with achalasia which can be worked up with barium swallow (or esophageal manometry)
  • Key idea: Dysphagia to solids AND LIQUIDS is due to problems with esophageal motility, whereas dysphagia with solids that later leads to dysphagia to liquids more associated with obstructing mass lesions (cancer, esophageal stricture, etc.) that should be worked up with endoscopy

5. A 14-month-old boy is brought for a well-child examination…

Reassurance

  • Key idea: Normal age for child to begin walking is between 12 and 15 months

6. A 67-year-old man comes to the physician because of a 1-month…

Randomized, controlled clinical trial

  • Key idea: Randomized, controlled clinical trial is the best type of research study for getting accurate/useful results

7. A 32-year-old woman is brought to the emergency department…

Triglycerides

  • Young woman with signs of pancreatitis (epigastric abdominal pain with nausea/vomiting) and elevated amylase, most consistent with acute pancreatitis
  • Diagnosis of acute pancreatitis requires 2/3: (1) Acute epigastric pain radiating to the back (2) Increased serum amylase or lipase to at least 3X upper limit of normal (3) Characteristic imaging findings
  • Key idea: Two most common causes of acute pancreatitis are gallstones and alcohol, so in this patient who had a cholecystectomy and does not use alcohol, we would think of other potential causes such as hypertriglyceridemia, hypercalcemia (this patient has normal calcium), steroids, autoimmune pancreatitis, post-ERCP, drugs (diuretics, IBD drugs, anticonvulsants, etc.)

8. An 18-year-old woman comes to the physician because of a…

Splenectomy

  • Young woman with a family history of anemia presents with jaundice, splenomegaly and anemia with normal reticulocyte count, negative Coombs test and elevated mean corpuscular hemoglobin concentration with a peripheral smear showing spherocytes (RBCs without area of central pallor), most consistent with hereditary spherocytosis
  • Key idea: Hereditary spherocytosis is primarily transmitted in an autosomal dominant fashion and is treated with splenectomy (because the hemolysis occurs in the spleen because the splenic macrophages recognize the spherocytes as being misshapen)

9. A 17-year-old girl comes to the physician because of a 4-month…

Pilosebaceous follicles

  • Adolescent with a polymorphic facial rash with comedones and inflammatory papules, most consistent with acne which is due to pathology within pilosebaceous follicles
  • Apocrine glands associated with hidradenitis suppurativa, epidermal-dermal junction associated with bullous pemphigoid and panniculitis associated with erythema nodosum

10. During examination prior to participation in school sports…

Repeat blood pressure measurement in 4 weeks

  • Key idea: In a patient presenting with hypertension (particularly a child), you always should start with a trial of lifestyle interventions (playing sports in this patient’s case) before thinking about using blood pressure medications

11. A previously healthy 24-year-old woman comes to the…

Decreased acethylcholine receptors

  • Presentation is most consistent with myasthenia gravis (young woman or older man presenting with dysphagia, dysarthria and eye weakness that is worse at the end of the day), which is caused by antibodies attacking acetylcholine receptors and leading to reduced acetylcholine receptors
  • Decreased release of Ach from motor nerve terminals = Lambert-Eaton syndrome
  • Degeneration of muscle fibers = Muscular dystrophy

12. A 37-year-old man comes to the physician because of progressive…

Demyelination of axons

  • Young woman with a recent URI who presents with ascending weakness, absent deep tendon reflexes and mildly decreased touch/vibration, most consistent with Guillain-Barre syndrome
  • Key idea: Although Guillain-Barre known for ascending weakness, it also commonly leads to facial paralysis and/or respiratory failure
  • Key idea: Guillain-Barre is an autoimmune disease seen following a URI or GI infection that leads to demyelination

13. Six hours after coronary artery bypass grafting, a 62-year-old…

Surgical exploration of the mediastinum

  • Patient <24 hours after CABG who develops hypotension, decreased urine output, decreased cardiac output and a widened mediastinum, concerning for mediastinal bleed
  • Key idea: Post-op patients with acute hemodynamic instability almost always will require surgical exploration/correction (especially in NBME questions)

14. A 72-year-old man comes to the physician because of a…

Muscle

  • Older patient on a statin drug who presents with muscle tenderness and objective muscle weakness, most consistent with a statin myopathy
  • Key idea: Important causes of a myopathy on NBME exam include glucocorticoids/Cushing’s, inflammatory myopathies (polymyositis, dermatomyositis), statin-induced myopathy and hypothyroid myopathy

15. A 42-year-old man comes to the emergency department at…

Discharge and encourage fluid intake

  • Young, otherwise healthy man presents with severe flank pain, CVA tenderness and hematuria with pain that is responsive to morphine, most consistent with kidney stone
  • Key idea: Patients with a kidney stone that is <10 mm and/or not causing urosepsis or renal failure can often be medically managed with hydration, pain control, alpha blockers and straining urine (to look for passage of stone)

16. A 62-year-old woman is admitted to the hospital because…

Intravenous fluid therapy

  • Middle-aged woman with CKD (likely due to type 2 diabetes) who presents with symptoms (fever, cough, abnormal CXR) concerning for pulmonary embolism vs community-acquired pneumonia (fever, CXR finding) who also has poor oral intake during her hospital stay; during the course of her hospital stay she receives a CT angiogram to work-up potential pulmonary embolism and is later found to have an acute kidney injury most likely due to contrast nephropathy (form of acute tubular necrosis) in the setting of low fluid intake
  • Key idea: Contrast nephropathy is a feared complication of imaging studies with contrast, particularly among patients with baseline renal insufficiency (such as this patient), and many studies have found that IV fluid administration prior to contrast administration can be useful for decreasing the risk of developing contrast nephropathy
  • https://www.ncbi.nlm.nih.gov/pmc/articles/PMC5954945/

17. A previously healthy 42-year-old woman has had generalized…

Thymoma

  • Young woman + weakness affecting the eyes + anterior mediastinal mass = myasthenia gravis secondary to thymoma
  • Key idea: Myasthenia gravis associated with thymoma, whereas Lambert-Eaton associated with small cell lung cancer

18. An 18-year-old primigravid woman at 37 weeks’ gestation is…

Amniotomy and vaginal delivery

  • Woman at 37 weeks’ gestation presenting in labor who has history of genital herpes but currently does not have lesions or prodromal symptoms and therefore can undergo vaginal delivery
  • Key idea: Women with history of HSV should begin acyclovir prophylaxis at 36 weeks gestation and women with active HSV lesions or prodromal symptoms during labor should have a Cesarean delivery performed

19. A county health officer investigates an outbreak of illness…

Inadequate refrigeration of the implicated food

  • Causes of rapid-onset food poison (< 6 hours) are due to ingestion of pre-formed toxin and are most commonly caused by Staph aureus (cream-based food such as egg salad) or Bacillus cereus (reheated rice syndrome)
  • Key idea: Staph aureus food poisoning (most likely in this case given egg salad and rapid-onset symptoms) can either be due to improper cooking or inadequate refrigeration, but in this case given the history of a family picnic and the food likely being left out all day, inadequate refrigeration is more likely

20. A 52-year-old man comes to the physician because of excruciating…

Indomethacin

  • Middle-aged man presents with acute onset, atraumatic pain of the great toe with swelling, erythema and tenderness of the metatarsophalangeal joint on exam, most consistent with acute gout
  • Key idea: Acute gout should be treated with NSAIDs (often indomethacin) and patients should NOT be started on chronic gout drugs (allopurinol, probenecid, etc.) until the acute flare has resolved because these drugs can lead to rapid shifts in uric acid levels that can exacerbate/cause a new flare
  • Note: Patients can sometimes be given oral or intra-articular steroids, but NSAIDs are tried first due to lower risk and good effectiveness in the majority of patients

21. A 22-year-old primigravid woman at term is admitted to the…

Urinary stasis

  • As the uterus enlarges, it can begin to press upon nearby structures, including the urethra –> Impaired bladder emptying –> urinary stasis –> nidus for infection

22. A 6-month-old boy is brought to the physician because of a…

Mupirocin ointment

  • Key idea: Honey-colored crust = Impetigo = Staph or Strep infection = Topical antibiotic treatment (mupirocin)
  • Irritant dermatitis (erythema in the groin sparing flexural creases) –> Petroleum jelly or zinc oxide
  • Candida dermatitis (beefy-red rash involving skinfolds and with satellite lesions) –> Topical antifungal therapy (like nystatin)

23. A 4-month-old infant is brought to the emergency department…

Child abuse

  • Common injuries highly associated with child abuse include retinal hemorrhages and intracranial injury (particularly subdural hematoma due to shearing of bridging veins with violent shaking)
  • Can also commonly see long-bone spiral fractures or rib fractures, but importantly the NBME will at times try to trick you by having a parent bring in an infant with a history incompatible with bone fractures who has osteogenesis imperfecta
  • Key idea: One of the other presentations is if the parent’s story does not fit known developmental milestones (saying that a 2 month old rolled off a table when children typically do not roll until ~4 months)

24. A homeless 66-year-old man is admitted to the hospital because…

Decreased calcium

  • Key idea: ON THE BOARDS, ALCOHOLISM = MALNUTRITION!
  • Hypomagnesemia –> Refractory hypokalemia and hypocalcemia
  • Key idea: In the setting of hypocalcemia, important to always check a magnesium level and a serum albumin level (low albumin –> low total calcium)

25. A 16-year-old girl is brought to the physician by her mother…

Family therapy

  • Family therapy is appropriate in this case because the father, mother and daughter could benefit from improved communication and coping strategies
  • Note: Family therapy is also effective in the management of a patient with schizophrenia (helps to educate family about symptoms, course, etc.)

26. A 47-year-old woman is brought to the emergency department…

Thoracic aorta

  • Patient involved in a significant motor vehicle accident with acceleration-deceleration forces who develops neck pain and chest pain with bruising over the sternum and a widened mediastinum (almost always associated with aortic dissection or aortic injury), most consistent with blunt thoracic aortic injury leading to incomplete aortic rupture
  • Key idea: Involves the thoracic aorta because the aortic isthmus is tethered by the ligamentum arteriosum and is therefore not freely mobile and is prone to developing severe stretching/shearing forces in the setting of an acceleration-deceleration injury
  • https://www.nejm.org/doi/full/10.1056/NEJMra0706159

27. A 52-year-old woman is brought to the physician by her…

Bipolar disorder

  • Patient with a history of episodes consistent with depression (sleep changes, decreased energy/interest) who now presents with irritability, decreased need for sleep, pressured speech and distractibility, all consistent with bipolar disorder
  • Bipolar disorder associated with DIG FAST symptoms (Distractibility, Impulsivity, Grandiosity, Flight of ideas, increased Activity, decreased Sleep and Talking loudly/rapidly)
  • Key idea: Aspects of psychosis (such as this patient’s auditory hallucinations) can be seen in multiple psychiatric conditions including bipolar disorder and are NOT specific to schizophrenia, schizophreniform, etc.

28. One day after removal of a large meningioma, a 42-year-old…

Diabetes insipidus

  • Patient with recent brain surgery who has developed hypernatremia with dilute urine and polyuria, consistent with diabetes insipidus (most likely central due to brain surgery)
  • Causes of central diabetes insipidus (decreased ADH release): Pituitary tumor, trauma, surgery, autoimmune disease, ischemic event
  • Causes of Nephrogenic diabetes insipidus (decreased responsiveness of kidney to ADH): Inherited, hypocalcemia, hypokalemia, lithium, demeclocycline
  • Note: Excessive ADH production and renal salt wasting would lead to hyponatremia, 0.9% is isotonic (so wouldn’t lead to significant electrolyte changes), and hyperaldosteronism leads to normal Na+ levels due to aldosterone escape

29. An asymptomatic 23-year-old man comes to the physician for…

Treatment of latent tuberculosis now

  • Patient with a positive PPD (>15 mm positive in all patients) who is asymptomatic and without CXR findings = Latent tuberculosis –> Treatment (especially on NBME exam)
  • Key idea: PPD induration (NOT erythema) required for positivity depends upon patient risk factors with >5 mm being positive in patients with significant immunosuppression (HIV, organ transplant, immunosuppressant meds), recent contact with patient with active TB or patients with CXR findings consistent with TB // >10 mm being positive for patients from Tb endemic countries, IVDU, residents of high-risk settings (prisons, nursing homes, homeless shelters, etc.), children < 4 years old and patients working in mycobacterial labs // >15 mm in all patients
  • Note: 3 treatment options for latent tuberculosis include (1) Isoniazid and rifapentine +/- pyridoxine (2) Isoniazid +/- pyridoxine (3) Rifampin

30. A 24-year-old man has shortness of breath 2 days after open…

Fat embolism syndrome

  • Key idea: Fat embolism often develops 24 to 72 hours after inciting event (long-bone fracture, orthopedic surgery, etc.)
  • Key idea: Fat embolism is a clinical diagnosis that leads to triad of (1) Respiratory distress (2) Neurologic dysfunction (3) Petechial rash (or thrombocytopenia)

31. A previously healthy 6-year-old boy is brought to the…

Operative procedure

  • Young child presenting with acute unilateral scrotal pain, nausea/vomiting, abdominal pain with involuntary guarding and a discolored, swollen hemiscrotum prompting concern for testicular torsion which should be taken directly to OR!
  • Key idea: Surgical correction should be performed within 6 hours in order to salvage the affected testicle and importantly torsion often occurs in the setting of an inherited defect in the attachment of the testicle within the scrotum that will affect both testicles, so an orchiopexy should be performed both on the affected and unaffected side

32. A 47-year-old woman comes to the physician because of…

Intravascular volume depletion

  • Middle-aged woman with potential infectious disease (fever) leading to one week of uncontrolled hyperglycemia who has signs of orthostatic hypotension (drop in systolic BP >20 or diastolic BP >10 when moving from sitting to standing) and is found to have glucosuria, most consistent with intravascular volume depletion due to glucose acting as an osmotic agent and leading to polyuria
  • Key idea: Infections often lead to increased insulin requirement by causing a reactionary hyperglycemia

33. A 50-year-old woman with a 5-year history of metastatic breast…

Pericardial window

  • Middle-aged patient with a history of metastatic breast cancer who presents with triad of hypotension, jugular venous distention and distant heart sounds who also has electrical alternans and pericardial effusion, all consistent with a diagnosis of pericardial tamponade often secondary to breast cancer metastasis to the pericardium
  • Key idea: Although cardiac tamponade is often confirmed with an echocardiogram, if your index of suspicion is high you would proceed directly to therapeutic pericardial window due to significant morbidity/mortality associated with condition

34. A 4-year-old boy has had increasing fatigue since a viral illness…

Bone marrow aspiration

  • Young child with a recent URI who is presenting with pancytopenia and lymphadenopathy and hepatomegaly, concerning for acute lymphoblastic leukemia
  • Key idea: Viral infections are known for causing pancytopenia, but in this patient who fits the correct demographic and has lymphadenopathy and hepatomegaly (signs of lymphoid hyperplasia) fit well with a diagnosis of acute lymphoblastic leukemia, which would have at least 20% blast cells on bone marrow aspiration

35. A 27-year-old woman comes to the physician because of a…

Small bowel obstrution

  • Young woman with Crohn’s disease who has severe abdominal pain, nausea/vomiting, and a distended tender abdomen, most consistent with small bowel obstruction likely due to a stricture in setting of Crohn’s disease
  • Key idea: Elevated amylase is NOT specific for pancreatitis and can also be seen in patient with vomiting
  • Key idea: Small bowel obstruction classically leads to 5 symptoms: (1) Obstipation (constipation, no flatulence) (2) High-pitched bowel sounds (bowel working hard to try to get past obstuction) (3) Abdominal distention (4) Abdominal pain (5) Nausea/vomiting
  • Crohn’s disease leads to transmural fibrosis and can therefore damage underlying stem cells and lead to fibrotic strictures through healing process

36. An 82-year-old woman with metastatic breast cancer who is…

Increase the frequency of dose to every 3 hours

  • Key idea: While physicians should be on the look-out for signs of opioid addiction in patients receiving pain medications, in patients with metastatic cancer to bone it is common for them to develop tolerance and for the pain to progress, so their requests for increased pain management are valid and should be managed through a palliative care lens
  • Mild cancer pain: Acetaminophen, NSAIDs
  • Moderate cancer pain: Weak opioids (codeine, hydrocodone, tramadol)
  • Severe cancer pain: Strong short-acting opioids (morphine, hydromorphone) –> Long-acting opioids (fentanyl, oxycodone) and short-acting opioids for breakthrough pain

37. Two days after beginning ACTH therapy for multiple sclerosis…

Haloperidol

  • Key idea: Systemic steroids are associated with psychosis and this patient is receiving ACTH, which will stimulate the adrenal gland to release increased amounts of glucocorticoids/steroids, likely precipitating her psychosis (delusions, bizarre behaviors, etc.)
  • Alprazolam = Benzo (avoided in older patients due to risk of delirium)
  • Amitriptyline = TCA antidepressant (refractory depression, neuropathic pain, migraines)
  • Lithium = Bipolar disorder
  • Sertraline = SSRI (depression, PTSD, panic disorder, etc.)

38. A 27-year-old man comes to the physician with his 27…

Bilateral varicoceles

  • Key idea: Varicoceles are a major cause of male-dependent infertility because the increased pooling of blood in the pampiniform plexus near the testes leads to increased temperature, which leads to impaired Sertoli cell function (Note: Leydig cells will still be functioning well so testosterone levels will be normal)
  • Key idea: In NBME-style infertility questions, if one of the partners has had a previous child then the problem likely involves the other individual

39. A previously healthy 4-year-old boy is brought to the physician…

Upper respiratory tract infection

  • Did not have access to media player, but the patient’s presentation is consistent with a URI
  • CHF: Cyanosis, delayed capillary refill, etc.
  • Idiopathic pulmonary hypertension: Lower extremity edema, congestive hepatopathy, loud P2, RV heave
  • Pericarditis: Febrile, friction rub, etc.
  • VSD: Often asymptomatic until later age, will lead to a systolic ejection murmur in the lower left sternal border

40. A previously healthy 26-year-old man comes to the…

Increased calcium

  • Young man presents with fever, erythema nodosum (painful, red, tender nodules on anterior shins) and bilateral hilar fullness, most consistent with sarcoidosis
  • Key idea: Sarcoidosis (like other forms of granulomatous disease) is associated with hypercalcemia because of increased activity of 1-alpha hydroxylase leading to increased production of activated vitamin D –> hypercalcemia, hyperphosphatemia
  • Ddx for erythema nodosum: Sarcoidosis, Strep infections, tuberculosis, inflammatory bowel disease, Behcet disease, endemic fungal disease (cocci, histo)
  • Key idea: If patient has erythema affecting both legs, then cellulitis is virtually ruled out! (bilateral cellulitis very rare)

41. A 4-year-old girl is brought to the physician because of cough…

T-lymphocyte dysfunction

  • Young child with non-reactive skin testing for multiple antigens (which is a type 4 hypersensitivity process dependent upon T cells), raising concern for T lymphocyte dysfunction
  • Key idea: Patients with HIV can have a falsely-negative PPD in the setting of M. tuberculosis because they have low T cell function and impaired Type 4 HS reaction
  • Antibody deficiency = Bruton’s agammaglobulinemia = Recurrent infections with encapsulated infections
  • Complement deficiency = Recurrent Neisseria infections
  • Impaired chemotaxis = Leukocyte adhesion deficiency = Recurrent abscesses without pus, delayed wound healing, delayed umbilical cord separation
  • Impaired respiratory burst = chronic granulomatous disease = Recurrent skin abscesses and infections with catalase positive organisms (Staph, Pseudomonas, Aspergillus, Nocardia, Serratia, etc.)
  • Splenic dysfunction (sickle cell disease, trauma patient) = Encapsulated infections (Strep pneumo, H. influenzae, Neisseria)

42. A 47-year-old woman comes to the physician because of fever…

CT scan of the head

  • Patient presenting with signs/symptoms concerning for meningitis vs. subarachnoid hemorrhage who also has signs of increased intracranial pressure and should therefore have a CT scan performed before a lumbar puncture
  • Key idea: Optic fundi cannot be visualized = Papilledema = Increased intracranial pressure = Contraindication to lumbar puncture due to risk of cerebral herniation
  • Key idea: Both meningitis and subarachnoid hemorrhage can lead to fever and signs of meningismus (pain with neck flexion, Kernig sign, etc.)

43. A 24-year-old woman comes to the physician because she has…

Osteoporosis

  • Anorexia nervosa –> Functional hypothalamic amenorrhea (also seen in extreme exercisers, professional athletes, etc.) –> Decreased GnRH release from hypothalamus –> Decreased LH and FSH release from anterior pituitary –> Decreased estrogen production by ovaries –> Increased osteoclast activity and decreased osteoblast activity –> Osteoporosis

44. A 19-year-old man is brought to the emergency department by…

PCP

  • Patients with PCP intoxication often have nystagmus, violent behavior and fluctuations between irritability and severely decreased activity (mute, motionless, etc.)
  • Key idea: On the NBME, drug intoxication + nystagmus = PCP use

45. An 82-year-old woman is brought to the emergency department…

Central retinal vein occlusion

  • Unilateral vision loss + Dilated retinal veins and widespread retinal hemorrhages on fundoscopy (“blood and thunder” appearance) = Central retinal vein occlusion
  • Central retinal artery occlusion = Cherry-red spot on fundoscopy
  • Corneal abrasion = Positive Fluorescein staining, sensitivity to light, pain
  • Glaucoma = Bilateral lens opacity and glare from lights
  • HSV conjunctivitis = Dendritic ulcers
  • Optic neuritis = Optic disk pallor

46. A 37-year-old woman has had malaise, muscle aches…

Culture for herpes simplex

  • Painful genital ulcers: (1) Herpes simplex virus: Small vesicles/ulcers on erythematous base with mild lymphadenopathy (2) Chancroid: Large, deep ulcers with a soft, friable base and severe suppurative lymphadenopathy
  • Painless genital ulcers: (1) Syphilis: Single ulcer with regular borders and hard base (chancre) (2) Lymphogranuloma venereum: Painless, shallow ulcers with painful fluctuant lymphadenopathy (buboes)

Exam section 2:

1: Over the past 2 years, a 67-year-old man has had gradually…

Abstinence from alcohol

  • Older man with significant smoking and drinking history who presents with signs of cerebellar dysfunction (ataxia, wide-based gait), most consistent with alcoholic cerebellar degeneration (degeneration of Purkinje cells in cerebellar vermis –> Truncal dyscoordination with impaired gait and postural incoordination with preserved limb coordination)

2. A 27-year-old male physician is inadvertently struck with a…

No treatment necessary

  • Patient is adequately vaccinated against hepatitis B (“adequate concentrations of antihepatitis B antibodies) and therefore does not need post-exposure prophylaxis
  • Key idea: If patient was unvaccinated or had unknown vaccination history, they would receive both hep B immunoglobulin and hepatitis B vaccine

3. A previously healthy 32-year-old woman at 16 weeks’ gestation…

Thrombotic thrombocytopenic purpura

  • Young pregnant woman presenting with fever, neurologic dysfunction, anemia with schistocytes (microangiopathic hemolytic anemia), and thrombocytopenia, associated with thrombotic thrombocytopenia purpura
  • Mnemonic for components of thrombotic thrombocytopenic purpura or hemolytic-uremic syndrome: Brain FART –> Brain (neuro symptoms, more associated with TTP), Fever, Anemia (hemolytic), Renal problems (more associated with HUS), Thrombocytopenia
  • Causes of micro/macroangiopathic hemolytic anemia (Anemia with schistocytes, unconjugated hyperbilirubinemia, etc.): DIC, TTP, HUS, SLE, HELLP syndrome, hypertensive emergency, aortic stenosis, prosthetic heart valves
  • Henoch-Schonlein purpura: SAG –> Skin (palpable purpura), Arthralgias and GI symptoms (with association with intussusception)

4. Six hours after undergoing left thoracostomy for a coarctation…

Analgesic therapy

  • Newborn who underwent major cardiac surgery who is fussy/irritable with mildly elevated pulse and mildly increased blood pressure, which is most likely due to inadequate pain management

5. A 43-year-old man comes to the physician because of a 3-day…

Haemophilus influenzae

  • Middle-aged man with significant smoking history and signs of pneumonia (fever, pleuritic chest pain, productive cough, shortness of breath, increased tactile fremitus and dullness to percussion) who has sputum gram stain showing gram-negative bacilli, most consistent with H. influenzae
  • Pseudomonas aeruginosa would also lead to gram-negative bacilli, but would be more common in a cystic fibrosis patient
  • N. meningitidus –> Gram-negative cocci
  • Strep pneumo –> Gram positive diplococci

6. A previously healthy 57-year-old man comes to the emergency…

Increased urinary excretion of calcium

  • Middle-aged man with signs/symptoms and CT scan consistent with renal calculi who has lab findings consistent with primary hyperparathyroidism (increased calcium, decreased phosphorous, increased PTH)
  • Key idea: Although elevated PTH levels will lead to increased reabsorption of calcium from the renal tubules, the amount of calcium being filtered by the kidney will still be relatively higher than normal due to increased calcium in the blood

7. A 67-year-old man comes to the emergency department because…

No cleaving of C-terminal peptides on angiotensin 1

  • Angiotensinogen –(renin)–> Angiotensin 1 –(angiotensin-converting enzyme)–> Angiotensin 2 –> Vasoconstriction, increased aldosterone, increased ADH, increased sympathetic activity
  • Key idea: Although patient on ACE inhibitor will have decreased mineralocorticoid production in the adrenal cortex, the decreased angiotensin 2 will have a larger overall effect

8. A 32-year-old man is brought to the emergency department…

Hypertrophic obstructive cardiomyopathy

  • Young man with exertional syncope who has a systolic ejection murmur at the left sternal border that increases with decreased preload –> HOCM
  • Key idea: Only 2 heart murmurs that increase in intensity with decreased preload (sitting to standing, diuretics, etc.) are HOCM (decreased preload –> increased obstruction) and Mitral valve prolapse (decreased preload –> less taut chordae tendinae –> earlier prolapse)

9. A 22-year-old man is brought to the emergency department…

Atropine

  • Patient exposed to gas who presents with signs of organophosphate poisoning (increased Ach leads to muscle fasciculations, increased sweating and increased parasympathetic effects (miosis, drooling, rhinorrhea, bronchoconstriction)
  • Organophosphates are acetylcholinesterase inhibitors –> Increased acetylcholine and therefore can be treated by using muscarinic antagonists (such as atropine)
  • Key idea: Atropine will block the muscarinic effects of increased acetylcholine, but will not block the nicotinic effects (muscle fasciculation –> muscle paralysis)
  • Atropine = Muscarinic antagonist –> Increased body temperature (decreased sweating), decreased parasympathetic functions (tachycardia, dry mouth, mydriasis, constipation, disorientation, urinary retention)

10. A 32-year-old woman comes to the physician because of…

Amoxicillin

  • Young patient with headache, arthritis, fever several weeks after a red rash who has IgM antibodies (indicator of recent infection) positive for Borrelia burgdorferi, most consistent with Lyme disease
  • Lyme disease often treated with doxycycline, but in pregnant women and children it is often treated with amoxicillin or cefuroxime (due to teratogenic effects of doxycycline)

11. A 57-year-old woman comes to the physician because of…

Osteoarthritis

  • Older woman without signs of inflammation (no fever, normal ESR) who has asymmetric groin pain (indicative of hip pathology) with an x-ray showing joint-space narrowing particularly between the left femoral head and acetabulum, consistent with osteoarthritis
  • Pathology in the hip –> Groin pain (HIGH YIELD!!!!)
  • Features of osteoarthritis –> Joint cysts/osteophyte, subchondral sclerosis, joint-space narrowing

12. A 54-year-old man comes to the physician because of weakness…

Carotid endarterectomy

  • Carotid endarterectomy should be considered in patients who are symptomatic (TIA or ischemic stroke in distribution of affected vessel) with high-grade carotid stenosis (70-99%) without relative contraindications (persistently disabling neurologic deficits, life expectancy < 5 years, etc.)
  • Key idea: Carotid endarterectomy does NOT reverse the damage from the stroke that has already occurred, but would decrease risk of further strokes/TIA
  • Key idea: All patients with carotid artery stenosis should be initially managed with intensive medical management (aspirin, statin, BP control) and lifestyle interventions (quitting smoking, weight loss, etc.)

13. An asymptomatic 22-year-old nulligravid woman comes to the…

Oral contraceptive therapy

  • Benefits of OCPs: Contraceptive, decreased endometrial and ovarian cancer, treatment of hyperandrogenism (PCOS, acne)
  • Risks of OCPs: DVTs, hypertension, hepatic adenoma, stroke, increased risk of cervical cancer
  • Key idea: No effective screening (imaging, lab test) is routinely available for ovarian cancer

14. A sexually active 24-year-old man has had increasingly…

Genital herpes

  • “Dew drops on rose petals” appearance = Herpes infection
  • Painful genital ulcers: (1) Herpes simplex virus: Small vesicles/ulcers on erythematous base with mild lymphadenopathy (2) Chancroid: Large, deep ulcers with a soft, friable base and severe suppurative lymphadenopathy

15. A 12-month-old boy is brought to the emergency department…

B lymphocyte

  • Young boy who has been infected with Strep pneumo, H. influenzae, and now is infected N. meningitidis, all of which are the classic encapsulated bacteria, most consistent with X-linked Bruton’s agammaglobulinemia
  • Key idea: Encapsulated bacteria (SHiN: Strep pneumo, H. Influenzae and Neisseria) are harder for our bodies to destroy and are therefore more dependent upon opsonization, which requires IgG (which will be absent in patients with defects in B lymphocytes because they can’t produce plasma cells and therefore can’t produce immunoglobulins)

16. A 12-month-old boy is brought to the physician because of a…

Neutrophil

  • Young boy with persistent Staph aureus skin abscesses and lung empyema (which is commonly caused by Staph aureus), most consistent with chronic granulomatous disease (X-linked disease most commonly)
  • Key idea: Patients with chronic granulomatous disease are preferentially infected by catalase positive organisms, which can be remembered with the mnemonic BELCH SPANS (Burkholderia cepacia, E. coli, Listeria, Candida, H. pylori, Staph aureus, Pseudomonas aeruginosa, Aspergillus, Nocardia, Serratia)

17. A previously healthy 57-year-old woman with hypertension is…

Botulism

  • Previously healthy middle-aged woman who had home-canned foods who presents with GI symptoms, ocular symptoms, and autonomic symptoms with normal deep tendon reflexes, most consistent with Botulism
  • Key idea: Adult-onset botulism associated with consumption of canned foods, whereas infant-onset botulism (floppy baby syndrome) associated with consumption of honey and exposure to dust (construction site, etc.)
  • Guillain-Barre syndrome –> Ascending weakness over course of weeks associated with sensory symptoms and loss of deep tendon reflexes
  • Myasthenia gravis –> Often does not present acutely, would not present with autonomic symptoms and history often includes weakness that worsens with continued use, associated with thymoma
  • Lambert-Eaton syndrome –> Often does not present acutely, presents with autonomic dysfunction and diminished/absent deep tendon reflexes, and improves with continued use, associated with small cell lung cancer

18. A 36-year-old woman, gravida 1, para 1, has had heavy vaginal…

Uterine atony

  • Woman with complicated/prolonged delivery with excessive postpartum bleeding who does not have retained placenta, genital tract trauma or history of coagulopathy, and therefore most likely has uterine atony (most common cause of postpartum bleeding and commonly seen after prolonged/difficult delivery)
  • Causes of post-delivery bleeding is 4 T’s: (1) aTony (most common) (2) Trauma (3) Thrombin (coagulopathy) (4) Tissue (retained placenta, etc.)

19. A 25-year-old primigravid woman at 27 weeks’ gestation is…

Intramuscular administration of betamethasone

  • Patient presenting with preterm labor < 32 weeks: (1) Betamethasone (for fetal lung development) (2) IV penicillin (if GBS positive or unknown) (3) Magnesium (neuroprotective for baby) (4) Tocolytic (to stop contractions)
  • Active labor: Contractions AND cervical changes
  • C-section would only be indicated if there were signs of intramniotic infection, fetal instability or maternal instability
  • Note: Patients should receive intravenous antibiotics rather than oral antibiotics in a patient who is GBS positive or GBS unknown

20. A previously healthy 27-year-old man is brought to the…

Vasovagal syncope

  • Young otherwise healthy man who experienced loss of conciousness preceded by prodromal symptoms (nausea, sweating, lightheadedness) in a stressful situation and was found to be bradycardic at the time of the loss of consciousness, most consistent with vasovagal syncope
  • Key idea: Vasovagal syncope is the most common cause of loss of consciousness, especially among otherwise healthy patients
  • Hypoglycemia can also lead to loss of consciousness among otherwise healthy patients, but would not likely be associated with bradycardia or prodrome

21. A 32-year-old nulligravid woman with polycystic ovarian syndrome…

Clomiphene

  • PCOS patient who does not wish to become pregnant: Dual estrogen-progestin OCPs
  • PCOS patient who wants to become pregnant: Clomiphene (antagonists at estrogen receptors in hypothalamus –> prevents normal feedback inhibition –> increased release of LH and FSH from pituitary –> stimulates ovulation)
  • Note: PCOS patients with all types of fertility goals would benefit from weight loss

22. An afebrile 32-year-old woman comes to the physician because…

Enterotoxic Escheria coli

  • Pathogens leading to non-inflammatory diarrhea with no fecal leukocytes: V. cholerae, ETEC, B. cereus, Staph aureus, Giardia, etc.
  • Staph aureus = Acute onset food poisoning that resolves within 24 hours
  • Shigella and Yersinia = Bloody diarrhea with fecal PMNs

23. A 37-year-old man with end-stage Duchenne muscular dystrophy…

Determine if the patient wants to discuss his decision with anyone else

  • Key idea: Normal for patients with terminal illness to feel sad about their situation, but also important to look out for full-blown major depressive disorder
  • Should not force patient to discuss his plans with family, but important to determine whether patient would like other important stakeholders to be involved in the discussion

24. A 57-year-old woman with factor XI and IgA deficiencies is…

Anaphylactic transfusion reaction

  • Key idea: IgA deficiency classically leads to Anaphylaxis to IgA-containing blood products + Atopy + Autoimmune disease + GI/sinopulmonary infections (IgA involved in mucosal immunity)
  • Anaphylactic transfusion reaction: Respiratory distress/wheezing, angioedema, hypotension, hives/urticaria
  • Urticarial transfusion reaction: Angioedema, wheezing

25. A previously healthy 4-year-old boy is brought to the emergency…

Supination of the forearm with the elbow in slight flexion

  • Young patient with atraumatic elbow pain holding extremity with elbow flexion and forearm in pronation with no obvious signs of injury, which is most consistent with a Radial head subluxation (Nursemaid’s elbow) which commonly occurs when young children have their arm pulled and the radial head gets trapped beneath the annular ligament
  • Key idea: Can be treated by hyperpronation of forearm or by supination of forearm with flexion of elbow

26. A 7-year-old boy is brought for an examination prior to a…

Viridans streptococci

  • Key idea: Patients with abnormal heart valves are prone to developing endocarditis during periods of bacteremia, with dental work and toothbrushing leading to transient bacteremia with Viridans streptococci
  • Key idea: Staph aureus (IVDU, bacteremia) leads to acute endocarditis (days), whereas Viridans streptococci (dental work) leads to subacute endocarditis (weeks)

27. A 50-year-old woman with a history of hypothyroidism has…

Atrophic gastritis

  • Middle-aged woman with history of autoimmune disease who presents with megaloblastic anemia + neurologic symptoms with an abnormal Schilling test corrected by administration of intrinsic factor, most consistent with Autoimmune gastritis (antibodies against parietal cells in the stomach lead to (1) impaired intrinsic factor production –> B12 deficiency (2) chronic inflammation –> chronic/atrophic gastritis
  • Key idea: B12 deficiency leads to subacute combined degeneration, which often leads to problems with strength, fine touch, vibration and balance/cerebellar functions
https://www.memorangapp.com/flashcards/130443/USMLERx/

28. A previously healthy 47-year-old man comes to the physician…

Fibrillation potentials in multiple muscles of multiple extremities

  • Young man with chronic weakness who has UMN lesions (Babinski sign) and LMN lesions (atrophy, fibrillations), most consistent with amyotrophic lateral sclerosis (ALS), which leads to fibrillation potential in multiple muscles on electromyography
  • Key idea: ALS often presents with weakness in hands, difficulty swallowing (like in this patient), and changes to voice

29. An asymptomatic 27-year-old woman who is HIV positive…

Pneumococcal vaccine

  • Additional vaccination requirements for patients with HIV include (1) Vaccination for hepatitis B unless they have documented immunity (2) Strep Pneumo PCV13 followed by the 23-valent PPSV23 8 weeks later and again in 5 years and at age 65 (3) Meningococcal vaccine with boosters every 5 years
  • Note: Patients should receive one Tdap vaccine as an adult followed by boosters every 10 years

30. An 11-year-old girl is brought to the physician by her parents…

Education about puberty for the child and parents

  • Key idea: Patient is displaying normal teenager behavior and her symptoms are not causing her distress or leading to societally-inappropriate behavior (causing fires, stealing, etc.)

31. Two hours after emergency repair of a perforated ulcer, a…

Decreased potassium

  • Key idea: Digoxin toxicity can lead to premature PVCs (along with other arrhythmias), with digoxin toxicity being more prevalent in setting of hypokalemia (less potassium to compete with digoxin for binding to the Na/K ATPase)

32. A 55-year-old woman with known metastatic breast cancer…

Increased calcium

  • Metastatic breast cancer –> Bone involvement –> Hypercalcemia –> Hypercalcemic crisis –> Oliguria/anuria and mental status changes (somnolence or coma)
  • Key idea: Hypercalcemic crisis often only seen when calcium levels are > 14 mg/dL, and calcium levels often do not reach levels > 14 mg/dL except for in setting of malignancy

33. A 52-year-old woman comes to the emergency department…

Endoscopic retrograde cholangiopancreatography

  • Middle-aged woman with recent lap chole presenting with five days of fever, jaundice, and RUQ tenderness with labs showing a leukocytosis with cholestatic pattern (Alkaline phosphatase >> AST), most concerning for acute cholangitis that needs to be rapidly evaluated with an ERCP (associated with significant morbidity/mortality)
  • Key idea: Acute cholangitis –> Fever, jaundice, RUQ pain (Charcot’s triad) + hypotension, altered mental status (Reynold’s pentad)
  • Key idea: Even though patient has had cholecystectomy, they can still develop acute cholangitis due to a residual stone in the common bile duct or a retained gallstone in the cystic duct stump

34. A 42-year-old woman comes to the physician because of…

Urethral diverticulum

  • Woman with urinary incontinence exclusively after voiding found to have a cystic, tender mass in the vagina and a low post-void residual volume, suggestive of a urethral diverticulum
  • Interstitial cystitis: Discomfort/pain in bladder + Urinary frequency/urgency
  • Vesicovaginal fistula: Continuous loss of urine with clear fluid found in the vagina
https://www.youtube.com/watch?v=OsvxoMl62q8

35. An 8-hour-old newborn develops jaundice and respiratory…

Rh incompatibility

  • Newborn is found to have unconjugated hyperbilirubinemia with anemia (consistent with hemolytic anemia) and signs of volume overload (hepatosplenomegaly, edema), most consistent with Rh-hemolytic disease
  • Potential causes of anasarca or hydrops fetalis: Rh-hemolytic disease, parvovirus/CMV infection of mother, hemoglobin Barts disease (severe form of alpha thalassemia), etc.
  • Key idea: ABO incompatibility will not present with as severe of a presentation because majority of maternal antibodies against other AABO blood groups are IgM and do not readily cross the placenta (contrast with acquired Rh-targeting antibodies in Rh-negative mother who has been exposed to Rh-positive blood which are IgG and can easily cross the placenta)

36. A 32-year-old man comes to the physician because of a 3-month…

Trial of omeprazole therapy

  • Key idea: Patients with symptoms of GERD who are under 50 years old with symptoms for <5 years, no cancer risk factors and no alarm symptoms (melena, hematemesis, weight loss, anemia, dysphagia/odynophagia, etc.) should receive a trial of PPI therapy
  • Key idea: Patients who are 50+ years old with symptoms for >5 years or cancer risk factors or alarm symptoms would first receive an endoscopy

37. Four days after undergoing a right hemicolectomy for cecal…

Wound dehiscence

  • Key idea: Wound dehiscence often occurs ~5 days post-op
  • Wound dehiscence: Surgical wound opens back up, leading to increased serosanguinous discharge without signs of infection –> Manage conservatively with regular dressing changes, limiting mobilization, coughing, etc.
  • Evisceration: Dehiscence with protrusion of abdominal organs (viscera) through opening –> SURGICAL EMERGENCY!!!

38. A 37-year-old man comes to the physician because of a 6-month…

Alveolar hypoventilation

  • PAO2 = 150 – (PaCO2/0.8) = 150 – (56/0.8) = 150 – 70 = 80
  • A-a gradient = 80 – 70 = 10 < 12 –> Normal A-a gradient consistent with hypoventilation or decreased FiO2 (high altitude
  • Key idea: Common causes of hypoventilation include neuromuscular weakness, drug intoxication (particularly opioids) and obesity hypoventilation syndrome (which is most likely the case here)

39. A 17-year-old boy is brought to the emergency department…

Naloxone

  • Young patient with known heroin use who presents with altered mental status and hypoventilation, most consistent with heroin overdose
  • Opioid overdose should be treated immediately with naloxone (short-acting opioid receptor antagonist), and use medications such as naltrexone (long-acting opioid antagonists), methadone (long-acting opiate) or buprenorphine (partial opioid agonist) to prevent relapse
  • Flumazenil –> Benzodiazepine overdose

40. A 37-year-old woman with alcoholism is admitted to the…

Acute respiratory distress syndrome

  • Patient with severe pancreatitis who develops hypoxia, hypotension with elevated right-sided pressures and normal left-sided pressures and a severely increased A-a gradient, most consistent with acute respiratory distress syndrome
  • PAO2 = (713 x 0.6) – (PaCO2/0.8) = 428 – (38/0.8) = 428 – 48 = 380 >> 12 –> Increased A-a gradient
  • Congestive heart failure –> Decreased cardiac index and increased PCWP
  • Causes of ARDS: Sepsis, aspiration, pneumonia, trauma, pancreatitis

41. A 15-year-old boy is brought to the physician by his mother…

Major depressive disorder

  • Patient with acute stressor who develops behavior changes with changes in sleep, energy, appetite, interest, cognitive dysfunction with an abnormal affect, most consistent with major depressive disorder
  • Depression –> Depressed mood and SIGECAPS (Sleep changes, decreased Interest, Guilt, decreased Energy, Cognitive dysfunction, Appetite changes, Psychomotor slowing, Suicidal

42. A 75-year-old man is brought to the emergency department…

Heat stroke

  • Heat stroke: Elevated temperature with CNS dysfunction, caused by impaired thermoregulation
  • Heat exhaustion: Elevated temperature with no CNS dysfunction, caused by inadequate salt and water replacement
  • Malignant hyperthermia –> Often seen in patient receiving anesthetic who develops hyperthermia and rigidity

43. A 32-year-old man with major depressive disorder has had…

Methamphetamine

  • Methamphetamine is associated with a sensation of insects crawling on the skin –> skin picking
  • Technical term is delusional parasitosis

44. A 62-year-old woman is admitted to the hospital for…

Renal ultrasonography

  • Middle-aged woman with a recent GU surgery who develops a significant AKI with a BUN:creatinine ratio < 15 with a bland urinalysis, most concerning for a post-renal AKI secondary to ureteral damage, which would be worked-up with a renal ultrasound looking for hydronephrosis
  • Key idea: GU surgeries in women are highly associated with damage to the ureters, which can lead to a post-renal AKI (VERY HIGH YIELD FOR NBME EXAMS!)

45. A 72-year-old man is admitted to the hospital because of shortness…

Diuretic therapy

  • Decompensated heart failure most likely diagnosis given demographics (older patient with history of cardiac ischemia) + bilateral crackles in lung fields + low cardiac index with an increased pulmonary artery occlusion pressure (which is synonymous with pulmonary capillary wedge pressure and is a marker of left atrial pressure
  • Decompensated heart failure –> Diuretic therapy to help pull off fluids (while being careful to not cause AKI)

46. A 17-year-old girl is brought to the emergency department…

Cricothyrotomy

  • Patient with massive facial trauma precluding standard tracheal intubation and gurgling noises on exam, who should have an airway established via a cricothyrotomy
  • Indications for cricothyrotomy: (1) Inability to intubate (2) Inability to ventilate (3) Inability to maintain O2 sat > 90% (4) Severe traumatic injury that prevents oral/nasal tracheal intubation
  • Key idea: Cricothyrotomy is most often used in patients who have experienced a traumatic injury

Exam section 3:

1: A 34-year-old woman has had pain in the left hemithorax…

Costochondritis

  • Patient with history of increased activity + localized pain to the anterior chest that worsens with movement, deep inspiration and cough + Tenderness to palpation of affected area = Costochondritis
  • Key idea: Self-limiting over a matter of weeks, but may persist for up to a year

2. A 72-year-old man is hospitalized because of dyspnea for 6…

Cor pulmonale

  • Patient with progressive dyspnea with signs of right-sided heart failure (JVD, ascites, pitting edema but no pulmonary edema) and a history of DVT/PE, most consistent with cor pulmonale secondary to Group 4 pulmonary hypertension
  • Group 1 pulmonary hypertension = Pulmonary arterial hypertension (pre-capillary) [familial, toxins, connective tissue disease, etc.]
  • Group 2 PH: Left-sided heart failure
  • Group 3 PH: Chronic lung disease (COPD, OSA, asthma, etc.)
  • Group 4 PH: Chronic thromboembolic disease
  • Group 5 PH: Other causes [anemia, sarcoidosis, etc.]
  • Aortic stenosis or Mitral regurgitation = Systolic murmur on exam
  • Ischemic heart disease = Chest pain vs heart failure (pulmonary edema on chest x-ray)
  • Viral cardiomyopathy = Acute dyspnea, chest pain, fever

3. For 12 hours, a 25-year-old woman, gravida 2, para 1, at 20 weeks’…

Measurement of serum amylase activity

  • Young woman with symptoms concerning for acute pancreatitis (severe epigastric pain radiating to the back with nausea/vomiting)
  • Key idea: Measurement of serum amylase/lipase preferable to CT scan in setting of clinical pancreatitis (severe epigastric pain radiating to the back) because there can be a delay in the appearance of imaging findings
  • Diagnosis of acute pancreatitis requires 2/3: (1) Acute epigastric pain radiating to the back (2) Increased serum amylase or lipase to at least 3X upper limit of normal (3) Characteristic imaging findings

4. A 21-year-old primigravid woman at 37 weeks’ gestation has had…

Measurement of platelet count

  • Patient with signs of pre-eclampsia (hypertension and proteinuria) who should have delivery of the baby (because patient at 37 weeks’ gestation) and work-up for other end-organ complications associated with severe pre-eclampsia (platelet count, BUN/creatinine, etc.)
  • Pre-eclampsia (uncomplicated –> delivery at 37 weeks) = New-onset elevated blood pressure at >20 weeks gestation AND Proteinuria (Urine protein:creatinine ratio > 0.3 or 24-hour urine collection > 300 mg) or signs of end-organ damage
  • Features that point to severe pre-eclampsia requiring delivery at 34 weeks: BP > 160/110, thrombocytopenia, pulmonary edema, visual/cerebral symptoms, increased creatinine, increased LFTs

5. A 32-year-old man comes to the physician with his wife because…

Acute stress disorder

  • Symptoms of Acute stress disorder or PTSD: Patient with exposure to life-threatening trauma who experiences HARD symptoms (Hyperarousal, Avoidance behaviors, Re-living experience, Disturbed mood)
  • Acute stress disorder: Symptoms for 3 days to 1 month
  • PTSD: Symptoms for >1 month

6. An 18-year-old woman has had fever, mild nonproductive cough…

Epstein-Barr virus infection

  • Patient with fever, malaise, exudative pharyngitis, splenomegaly and prominent cervical lymphadenopathy, most consistent with infectious mononucleosis (EBV > CMV)
  • Strep throat would not last for 5 weeks
  • Diphtheria –> Pseudomembranes often in an immigrant patient

7. A 23-year-old man is brought to the physician by his mother…

Brief psychotic disorder

  • Patient who experienced a stressful event 1 week ago who has experienced auditory hallucinations of a somebody who was not killed, most consistent with brief psychotic disorder
  • Key idea: Bereavement can lead to auditory hallucinations from the decesased, but should not lead to such a decline in functioning as is seen in this vignette
  • Symptoms of psychosis: Delusions, hallucinations, disorganized speech, disorganized/catatonic behavior, negative symptoms (flat affect, etc.)
  • Brief psychotic disorder: At least 1 positive symptom for less than 1 month (often associated with stress)
  • Schizophreniform disorder: At least 2 psychosis symptoms for 1 month with decline in functioning between 1 month to 6 months
  • Schizophrenia: At least 2 psychosis symptoms for 1 month with decline in functioning for at least 6 months

8. A 72-year-old man comes to the physician for a follow-up…

Psychogenic polydipsia

  • Older patient who has been drinking 12-15 glasses of water daily and has a history of schizoaffective disorder found to have hyponatremia with low urine sodium and low urine osmolality, most consistent with psychogenic polydipsia
  • Key idea: Despite UWorld claims, urine osmolality does NOT need to be below 100 to diagnose psychogenic polydipsia
  • Key idea: Antipsychotics can commonly lead to dry mouth (as is seen in this patient), making a patient feel thirsty

9. A 27-year-old primigravid woman at 39 weeks’ gestation is…

Cesarean delivery

  • Category 3 fetal heart tracings (require immediate delivery): (1) Recurrent late decelerations + absent variability (2) Recurrent variable decelerations + absent variability (3) Bradycardia + absent variability (4) Sinusoidal pattern
  • Cervical dilation < 10 cm and need for immediate delivery –> C-section
  • Cervical dilation of 10 cm and need for immediate delivery –> Operative vaginal delivery (forceps delivery)
  • Key idea: Indications for C-section in the active phase (6-10 cm cervix) of labor (1) Active phase arrest with no cervical change for 4 hours with adequate contractions (>200 Montivideo units) or for 6 hours with inadequate contractions (2) Category 3 fetal HR tracing

10. A 37-year-old woman comes to the physician because of a…

Bromocriptine therapy

  • Young woman with irregular menses, galactorrhea, bitemporal hemianopsia found to have elevated prolactin and a pituitary microadenoma, most consistent with a prolactinoma
  • Key idea: Prolactin release from the anterior pituitary is inhibited by endogenous dopamine release from the hypothalamus, so we give patients with a prolactinoma dopamine agonists (bromocriptine, cabergoline) to inhibit prolactin release
  • Key idea: Patients with a macroprolactinoma (>10 mm) or symptomatic microprolactinoma should be started on dopamine agonists and should get a resection if the tumor is very large (>3 cm) or if the mass increases in size while on a dopamine agonist

11. A 32-year-old woman with asthma comes to the physician…

Allergic bronchopulmonary aspergillosis

  • Allergic bronchopulmonary aspergillosis: Hypersensitivity response associated with asthma and cystic fibrosis that can lead to bronchiectasis and eosinophils
  • Bronchiectasis = Chronic productive cough + linear atelectasis + thickened airways and irregular cystic opacities
  • Key idea: Treatment is months of treatment with itraconazole + oral corticosteroids
  • Cystic fibrosis –> Also associated with bronchiectasis and ABPA, but patient would have history of recurrent sinopulmonary infections, fat soluble vitamin deficiency, etc.
  • Alpha-1 antitrypsin deficiency –> Lungs and liver problems with family history

12. A previously healthy 62-year-old man is brought to the…

Intravenous labetalol

  • Older male patient with tearing chest pain radiating to the pack, different upper extremity blood pressures, aortic regurgitation murmur and a widened mediastinum on CXR, all of which are consistent with aortic dissection
  • Key idea: Beta-blockers (such as labetalol) are particularly useful in setting of aortic dissection because the force of blood being ejected from the LV and hitting the aorta can further propagate the tear, with beta blockers leading to reduced contractility and reduced force of blood ejection against the aorta

13. A 47-year-old man comes to the physician because of a 9-month…

Colonoscopy

  • Key idea: Patients get their first screening colonoscopy at 50 years old, so in this patient with bowel changes (constipation) and anemia (which cannot be fully explained by hemorrhoids), the next best step would be a colonoscopy

14. A 37-year-old woman, gravida 3, para 3, comes for a routine…

Bartholin duct cyst

  • Location of mass most consistent with a bartholin duct cyst
  • Key idea: Always try to envision anatomic position when given anatomical terms such as anterior, posterior, etc.
https://www.mayoclinic.org/diseases-conditions/bartholin-cyst/symptoms-causes/syc-20369976

15. An 8-year-old boy is brought to the physician 1 hour after the…

C1 esterase inhibitor

  • Young patient with recurrent angioedema WITHOUT urticaria with a strong family history, most consistent with hereditary angioedema which is caused by C1 inhibitor deficiency leading to impaired bradykinin breakdown, with increased bradykinin –> angioedema (similar pathophysiology to ACE-inhibitor angioedema)
  • Key idea: Autosomal dominant inheritance pattern
  • Key idea: These patients also commonly present with colicky abdominal pain and GI symptoms

16. An 18-year-old woman has had fever for 12 hours and obtundation…

Meningococcemia

  • Young woman who has been in a confined space with others (summer camp, college, etc.) who presents with fever, altered mental status, hypotension and purpuric skin lesions, most consistent with meningococcemia
  • Toxic shock syndrome –> Diffuse erythroderma
  • Lyme disease –> Erythema chronica migrans (“bulls-eye rash”)

17. Three days after open reduction and internal fixation of a right…

Duplex scan

  • Patient with recent surgery who presents with tightness/tenderness of the left calf, most concerning for a DVT (which should be worked-up with a duplex ultrasound)
https://en.wikipedia.org/wiki/Virchow%27s_triad

18. A 15-year-old girl is brought to the physician because of finger…

Systemic lupus erythematous

  • Young female patient with arthritis, malar rash, decreased hemoglobin and WBC count, low complement, positive ANA and urinalysis consistent with a glomerulonephritis, most consistent with systemic lupus erythematous
  • Mixed connective tissue disease: Autoimmune disease with variable features of SLE, systemic sclerosis and polymyositis that is characterized by Raynaud phenomenon, dactylitis, arthritis and inflammatory myopathy +/- other features

19. A 19-year-old African American man comes to the physician…

Digestive enzyme deficiency

  • Young African-American man with abdominal cramps/bloating with intermittent watery diarrhea that has been worse since starting college (which likely came with change in diet), most consistent with lactose intolerance
  • Key idea: Lactose intolerance is common in non-White populations (lactase persistence more common in Northern European populations who historically eat a lot of dairy products)

20. A previously healthy 82-year-old woman comes to the physician…

Brisk rotatory nystagmus on left lateral gaze

  • Physiological nystagmus: Alternating smooth pursuit in one direction and saccadic movement in the other direction (WOULD NOT BE ROTATORY)
  • All other answer changes represent normal changes in aging

21. A 16-year-old girl is brought to the emergency department after…

Aspirin

  • Patient who attempted an overdose who has a mixed anion-gap metabolic acidosis (140 – 104 – 6 = 30 > 12) with a significant respiratory alkalosis, most consistent with aspirin/salicylate poisoning

22. Four CAGE questions are used to screen for alcoholism...

Sensitivity: Increased // Specificity: Decreased

  • Number of yes answers needed for positive test moves from 2 –> 1 will lead to decreased false negatives (increased sensitivity) and increased false positives (decreased specificity)
  • Key idea: Low false negative –> high sensitivity (of patients with disease, how many test positive), whereas low false positive –> high specificity (of patients without disease, how many test negative)

23. Two days after admission to the hospital for detoxification…

Vitamin B1 (thiamine)

  • Patient with alcoholism presenting with ataxia (wide-based ataxic gait) and ophthalmoplegia (abnormal eye movements), most consistent with Wernicke encephalopathy (thiamine deficiency)
  • Key idea: Alcoholism = Malnutrition (at least on NBME)
  • Wernicke encephalopathy: Confusion, ataxia, ophthalmoplegia

24. A previously healthy 14-year-old boy is brought to the physician…

Repeated microfracture at the tendon insertion

  • Adolescent boy with knee pain exacerbated by jumping who has tenderness/swelling specifically over the tibial tubercle, most consistent with Osgood-Schlatter disease (patellar tendon pulls on tibial tubercle repeatedly, leading to inflammation and repeated microfractures)
https://kidshealth.org/en/parents/osgood.html

25. A 37-year-old woman with sickle cell disease comes to the…

Acute cholecystitis

  • Patient with sickle cell disease presenting with fever, RUQ pain, mild scleral icterus and positive Murphy sign (specific for acute cholecystitis on NBME exams) with an ultrasound showing cholelithiasis and pericholecystic fluid, most consistent with acute cholecystitis
  • Key idea: Acute cholecystitis classically does NOT lead to jaundice or hyperbilirubinemia, but this patient has another reason to have jaundice because sickle cell disease –> chronic hemolysis –> unconjugated hyperbilirubinemia
  • Risk factors for cholesterol gallstones: Female, Fat, Forty, Fertile
  • Risk factors for bilirubin gallstones –> Chronic hemolysis
  • Cholangitis –> Negative Murphy sign, signs of cholestasis (alkaline phosphatase > ALT/AST), dilated common bile duct

26. Three hours after undergoing emergency repair of a leaking…

Cardiogenic shock

  • Patient with a major aortic injury who presents with decreased cardiac index, hypotension, increased pulmonary capillary wedge pressure, increased CVP and increased PA pressure with ECG changes most consistent with cardiogenic shock
  • Distributive shock (sepsis, anaphylaxis) –> Increased cardiac index, decreased PCWP, decreased SVR
  • Hypovolemic shock –> Decreased cardiac index, decreased PCWP, increased SVR
  • CHF –> Pulmonary edema, lower extremity edema, elevated JVP, etc.

27. An otherwise healthy 70-year-old woman comes to the physician…

Paget disease of the breast

  • ~50% of women with Paget disease of the breast will have a negative mammogram and no masses on physical exam
  • Key idea: If patient had eczema, we would expect a longer history of eczema and rashes in places other than just the nipple

28. An otherwise healthy 40-year-old woman comes to the physician…

Physiologic discharge

  • Physiologic discharge: Non-spontaneous, non-bloody and minimal discharge with normal physical exam and mammogram
  • Pathologic discharge: Unilateral discharge, bloody/serous discharge and/or palpable lump or skin changes

29. A 27-year-old man comes to the physician because of a 2-month…

HIV antibody testing

  • Patient with disseminated molluscum contagiosum, which in an adult should prompt concern for immunodeficiency, most likely HIV
  • Key idea: Increased severity/dissemination of various skin conditions (molluscum contagiosum, psoriasis, tinea, seborrheic dermatitis, etc.) can be a sign of immunodeficiency and should be worked up with HIV testing

30. A 72-year-old man comes to the physician because of progressive…

Aortic stenosis

  • Old patient with “parvus et tardus” (diminished carotid upstrokes) and a 4/6 systolic murmur heard throughout the precordium with radiation to the carotids, most consistent with aortic stenosis
  • Key idea: Aortic stenosis can lead to heart failure because the LV will need to pump against increased afterload and will eventually become weak
  • Key idea: Most common cause of aortic stenosis is aortic valve calcification that is seen in elderly patients and developed over time

31. A 72-year-old man has decreased urine output 2 days after…

Blood: 1+ // Protein: 1+ // RBC: 0-5 // WBC: 0-5 // Casts: Pigmented granular // Other Microscopic findings: Renal tubular epithelial cells

  • Elderly patient with gram-negative bacteremia receiving gentamicin (aminoglycosides associated with acute tubular necrosis) who has developed oliguria and an AKI, most consistent with acute tubular necrosis
  • Acute tubular necrosis leads to pigmented granular casts
  • Causes of acute tubular necrosis: Sepsis, Aminoglycosides, ischemia/hypotension (spectrum of disease of pre-renal AKI), heavy metals, radiocontrast, ethylene glycol, rhabdomyolysis

32. A 42-year-old man has had a pruritic rash on his back for 4 days…

Cutaneous larva migrans

  • Patient who commonly has to maneuver on the ground while on his back who presents with a pruritic serpiginous rash on his back and is found to have an eosinophilia, most consistent with cutaneous larva migrans (caused by Ancyclostoma or Necator hookworks)
  • Ascariasis –> Bowel obstruction, biliary obstruction, eosinophilia
  • Scabies –> Pruritic rash worse at night with burrows in web-spaces of fingers/toes
https://dermnetnz.org/topics/cutaneous-larva-migrans/

33. A 72-year-old woman comes to the physician because of an increase…

Exploratory laparotomy

  • Older woman with history of breast cancer found to have increasing abdominal girth, ascites and a fixed non-tender adnexal mass most concerning for ovarian cancer
  • Key idea: Advanced ovarian cancer often spreads to the abdominal cavity, so exploratory laparotomy with cancer resection and inspection is needed for surgical staging
  • Key idea: Image-guided biopsy is contraindicated, as it can predispose to abdominal cavity seeding
  • Key idea: Ascites in a postmenopausal woman is always pathologic and is the origin of the typical symptoms of early satiety, weight gain, etc. seen in advanced ovarian cancer

34. Three days after hospitalization for treatment of severe muscle weakness…

Elevation of the head of the bed

  • Patient with severe muscle weakness who has developed fever, hypoxia, new infiltrates in the middle/lower lobes and a polymicrobial infection via bronchoscopy, most consistent with aspiration pneumonia

35. A 67-year old man comes to the physician because of easy fatigability…

Streptococcus pneumoniae

  • Older patient with multiple myeloma (monoclonal spike on SPEP and bone marrow biopsy showing >50% plasma cells), which leads to impaired production of normal immunoglobulins, leading to increased risk for sinopulmonary infections (due to impaired IgA production) and encapsulated infections (due to impaired IgG –> impaired opsonization –> increased risk of encapsulated infections such as Strep pneumo, H. influenzae and Neisseria)

36. A 24-year-old primigravid woman at 38 weeks’ gestation is…

Intravenous penicillin G

  • Indications for intrapartum prophylaxis against GBS infection: (1) GBS-positive rectovaginal culture during current pregnancy (2) GBS bacteriuria or GBS UTI in current pregnancy (3) Unknown GBS status + <37 weeks gestation OR intrapartum fever OR rupture of membranes for >18 hours
  • GBS with no allergy: IV penicillin
  • GBS with mild penicillin allergy (prior maculopapular rash without pruritis): Cefazolin
  • GBS with severe penicillin allergy (high risk of anaphylaxis, respiratory distress and urticaria): Clindamycin/erythromycin

37. A 14-year-old boy who has sickle cell trait is brought to the emergency…

X-ray of the left hip

  • Young patient with sickle cell trait presenting with 1-day of atraumatic thigh and knee pain with limited hip range of motion, concerning for avascular necrosis of the left hip
  • Causes of avascular necrosis: CASTS Bent LEGS (Corticosteroids, Alcohol, Sickle cell disease, Trauma, SLE, “the Bends”, LEgg-Calve-Perthes disease, Gaucher disease, SCFE

38. A 77-year-old man is brought to the physician by family members…

Hypothyroidism

  • Older patient with dementia, decreased DTRs, non-pitting edema of the lower extremities and slow movements, most consistent with hypothyroidism
  • Huntington disease –> Dementia, depression, chorea
  • Multi-infarct (vascular) dementia –> Step-wise dementia with focal neurologic defects (weakness, sensation, etc.)
  • Pernicious anemia –> Atrophic gastritis + B12 deficiency (lower extremity weakness, paresthesias, sensory disturbances, etc.)
  • Syphilis –> Lymphadenopathy, diffuse rash involving the palms and soles

39. A 42-year-old man is scheduled to undergo an MRI for evaluation…

Lorazepam therapy

  • Patient who developed symptoms of a panic attack while in an MRI (which involves being placed into a tight space and can lead to severe claustrophobia) with no history of previous illicit drug use or addiction, who could benefit from an acute-acting benzodiazepine for relaxation

40. Five days after undergoing an open splenectomy for immune…

X-rays of the chest

  • Patient with a recent surgery who develops dyspnea, decreased localized breath sounds, and a leukocytosis most concerning for a hospital-acquired pneumonia, which should be worked-up with chest x-ray
  • Pulmonary angiography (pulmonary embolism): Tachycardia, signs of DVT, hemoptysis, etc.

41. A 62-year-old man comes to the physician because of intermittent…

Colonoscopy to the cecum

  • Key idea: Important to rule out multiple masses, as that could affect surgical approach/management

42. A previously healthy 37-year-old woman comes to the physician…

Peristalsis: Decreased // Lower esophageal sphincter tone: increased

  • Previously healthy young woman with dysphagia to solids and liquids with upper endoscopy showing retained secretions in the esophagus with barium swallow showing “bird’s beak sign”, all of which is most consistent with achalasia
  • Key idea: Achalasia characterized by absence of esophageal peristalsis and impaired relaxation of lower esophageal sphincter

43. Three days after undergoing total hip replacement, a 50-year-old…

Epidural hematoma

  • Patient who recently had an epidural catheter removed who has developed inability to move his legs and urinary incontinence, most consistent with an epidural hematoma –> spinal cord compression –> lower extremity weakness and overflow incontinence due to impaired bladder contraction

44. A 67-year-old man comes to the physician because of aching…

Femoropopliteal arteries

  • Femoropopliteal artery plaque –> Calf claudication
  • External iliac artery plaque –> Thigh and calf claudication
  • Aortoiliac plaque –> Gluteal, thigh and calf claudication; impotence

45. A 37-year-old man comes to the physician because of persistent…

Calcium disodium edetate

  • Young patient who renovates old houses who presents with abdominal pain, wrist drop, sensory defects and a microcytic hypochromic anemia, most consistent with lead poisoning
  • Symptoms for lead poisoning: LEAD (Lead lines on gingivae and metaphyses of long bones, Encephalopathy and Erythrocyte basophilic stippling, Abdominal pain and sideroblastic Anemia and wrist Drop (with other neuro symptoms)
  • Lead poisoning treatments: Dimercaprol, EDTA (calcium sodium edetate = calcium sodium EDTA), succimer
  • Key idea: Potential buzzwords that should trigger thought of lead poisoning include battery factory, homemade whiskey, renovating old house, etc.

46. An otherwise healthy 42-year-old woman comes to the physician…

Reassurance

  • Otherwise healthy woman with an atraumatic, asymptomatic red eye upon waking, most consistent with subconjunctival hemorrhage
  • Key idea: Often results due to simple trauma from rubbing eyes vigorously and violent coughing spells, but on NBME often apparent upon waking
  • Key idea: Managed by observation because it typically resolves within 24-48 hours

Exam section 4:

1: An 82-year-old man with congestive heart failure (CHF) comes to the physician…

Decreased renal blood flow

  • CHF patient with increased diuretic dosage –> Too much fluid is pulled off –> Decreased effective circulating volume –> AKI (pre-renal, acute tubular necrosis)
  • Key idea: When taking care of a patient with decompensated heart failure, important to balance correcting volume overload with risk of causing an AKI

2. A previously healthy 68-year-old man is brought to the emergency department…

Acute myocardial infarction

  • Older man presenting with acute substernal chest pain radiating to his left arm (classic for MI), diaphoresis and an EKG showing ST elevation in leads 1, V2-V6, consistent with an anterior MI (left anterior descending involvement)

3. A 70-year-old woman is brought to the emergency department 3 hours…

Ventricular tachycardia

  • Older woman with known CAD who presents with acute chest pain, weakness and dyspnea who is found to be hypotensive with an ECG with a rate of 150 (2 big boxes b/w QRS complexes) and a wide QRS (>120 ms), most consistent with ventricular tachycardia likely secondary to coronary ischemia

4. A 52-year-old woman with a 2-year history of type 2 diabetes mellitus comes…

Add lisinopril to the medication regimen

  • Key idea: ACE inhibitors or ARBs should be initiated at onset of microalbuminuria in all diabetic patients
  • Key idea: Two most common causes of CKD include hypertension and diabetes, with diabetes leading to nephropathy because it leads to preferential arteriosclerosis of the efferent arteriole > afferent arteriole, thus leading to high pressure in glomerulus, resulting elevated GFR and damage to glomerulus over time (ACE inhibitor or ARB can reduce this damage by dilating the efferent arteriole, leading to a decreased GFR)
  • Key idea: Typical HbA1c goal for patients with diabetes is <7.0% (and therefore in this patient no need to switch diabetes meds)

5. A 20-year-old man is brought to the physician by his parents because of…

Schizophrenia

  • Young male patient (perfect demographic for onset of schizophrenia) who presents with 1 year of auditory hallucinations and negative symptoms, most consistent with schizophrenia
  • TSH is normal, so patient would not have psychosis secondary to thyroid dysfunction // Urine toxicology is normal so would not be substance-induced psychotic disorder
  • Brief psychotic disorder: 3 days to 1 month
  • Schizophreniform disorder: 1 month to 6 months
  • Schizophrenia: >6 months

6. A 32-year-old man is brought to the emergency department by a friend…

Dopamine

  • Young patient with recent medication addition to treat hallucinations (most likely an antipsychotic), who has fever, encephalopathy, unstable vitals (tachycardia, hypertension), elevated CK, and muscle rigidity, all of which are most consistent with neuroleptic malignant syndrome (secondary to antipsychotics that are dopamine antagonists)
  • Serotonin syndrome: 3 A’s (Autonomic dysregulation, Agitation and increased Activity (hyperreflexia, hypertonia, tremor, etc.) in a patient taking any drug that increases serotonin (SSRIs, SNRIs, TCAs, MAO inhibitors, triptans, MDMA, dextromethorphan, meperidine, St. John’s wort, ondansetron, tramadol, linezolid, etc.)

7. A 2-month-old girl is brought for a routine health maintenance examination…

Give all recommended immunizations for age

  • Key idea: Premature infants receive immunizations based on chronologic age (age since birth), with key exception being that they do not receive hepatitis B vaccine until they are at least 2 kg in weight
  • Key idea: Children with low-grade fever (<101 F) or mild illness (cold, runny nose, otitis media, mild diarrhea) can still get vaccines

8. A previously healthy 10-year-old girl is brought to the physician because…

No pharmacotherapy is indicated at this time

  • Previously healthy child with 2 day history of a “slapped cheek” rash that has spread to the reset of the body, low-grade fever and malaise, most consistent with Parvovirus B19 which is not treated with medication
  • Key idea: Although Parvovirus most associated with “slapped cheek” rash, it also can lead to reticular, erythematous rash on their extremities and trunk
  • Reminder: Parvovirus B19 associated with development of aplastic anemia

9. An asymptomatic 60-year-old man comes for a follow-up examination 2 months…

Observation

  • Patient who is asymptomatic with a hiatal hernia that can be managed with observation
  • Key idea: Patients with a symptomatic hiatal hernia (GERD), should be treated with PPIs or H2 blockers

10. A 22-year-old college student is brought to the student health clinic…

Olanzapine therapy

  • Young male patient (classic demographic for schizophrenia) with history of marijuana use (which increases risk for development of schizophrenia) who presents with 1-month of delusions (contaminated food), auditory hallucinations and impaired functioning (school performance deteriorated), most consistent with schizophreniform disorder –> Treat with antipsychotic (of which olanzapine is the only one listed)

11. A 32-year-old woman is brought to the emergency department because of…

Appendicitis

  • Young woman with 2-day history of fever, nausea/vomiting, RLQ abdominal pain with rebound and a neutrophilic leukocytosis, most consistent with acute appendicitis
  • Negative ultrasound for adnexal mass –> Not adnexal torsion, corpus luteum cyst, degenerating leiomyoma uteri, tubo-ovarian abscess, etc.

12. An unconscious 22-year-old man is brought to the emergency department…

Intravenous 0.9% saline

  • Patient engaging in intense physical activity presenting with signs of heat stroke (fever, seizure/encephalopathy, dry hot skin) and rhabdomyolysis (myoglobinuria), who should be treated with IV fluids to decrease the risk of myoglobin pigment stones precipitating kidney damage
  • Key idea: The major tenant of rhabdomyolysis management is aggressive IV fluid resuscitation because patients develop decreased effective circulating volume (due to fluid shifts into damaged muscle) and because patients are at risk for acute tubular necrosis secondary to myoglobinuria (with increased fluids helping to dilute these pigments and decrease the risk of tubular obstruction)

13. A 57-year-old man comes to the physician because of generalized weakness..

Interferon-alfa

  • Patient with signs/symptoms of mixed cryoglobulinemia (weakness, arthralgias, palpable purpura, positive cryoglobulins on labs) most likely secondary to hepatitis C contracted during blood transfusion (positive Hep C viral RNA), and therefore the patient should receive interferon-alpha therapy against the hepatitis C
  • Key idea: This test question is a bit outdated, with hepatitis C now being treated with combination of NS5A/B inhibitors (ledipasvir, velpatasvir, sofosbuvir, etc.) and NS3/4A inhibitors (simeprevir, etc.)
  • Key idea: To confirm diagnosis of hepatitis C, antibody testing needs to be followed up by viral RNA because hepatitis C will be spontaneously cleared by the body in 15-25% (no chronic Hep C)

14. A previously healthy 27-year-old African American woman, gravida 3…

Abruptio placentae

  • Woman at 34 weeks’ gestation with abdominal pain and vaginal bleeding (which should sound off alarm bells for placental abruption), most consistent with placental abruption
  • Anterior placenta and abdominal pain are inconsistent with placenta previa
  • Preeclampsia can lead to abruptio placenta, and that may be the case here
  • Risk factors for placental abruption: Maternal hypertension/pre-eclampsia/eclampsia, abdominal trauma, prior placental abruption, drug use (cocaine, tobacco)
  • Management of placental abruption: Aggressive maternal fluid resuscitation with crystalloids + delivery

15. A 16-year-old girl has had increasingly severe abdominal pain for 2 days…

Pelvic inflammatory disease

  • Adolescent girl who is sexually active with inconsistent contraceptive use who presents with fever, pharyngeal erythema and tenderness in the RLQ with cervical motion tenderness and a negative pregnancy test, most consistent with pelvic inflammatory disease (with likely gonorrhea co-infection of the throat)
  • Cervical motion tenderness: Pelvic inflammatory disease or ectopic pregnancy (negative pregnancy test rules it out)
  • Disseminated gonococcemia: Dermatitis, oligoarthritis, and enthesitis OR monoarticular septic arthritis

16. A 52-year-old woman comes to the physician because of a 2-week history…

Pleural metastases

  • Middle aged woman with history of breast cancer presenting with subacute dyspnea on exertion with dullness to percussion over the lower half of the right lung (consistent with pleural effusion) with no other sides of volume overload, most consistent with pleural effusion secondary to metastatic carcinoma
  • Pericardial tamponade: Hypotension, jugular venous distention, distant heart sounds, pulsus paradoxus
  • RLL pneumonia: Fever, cough, etc.

17. A 56-year-old woman with short-bowel syndrome caused by mesenteric…

Zinc

  • Signs of zinc deficiency: Alopecia, dermatitis, scaly skin around mouth/eyes, abnormal taste, impaired wound healing
  • Signs of copper deficiency: Hematologic abnormalities (microcytic anemia, leukopenia) and myeloneuropathy
  • Iron deficiency: Microcytic anemia

18. A previously healthy 18-year-old woman comes to the physician because…

Increased serum lactate dehydrogenase activity

  • Young previously healthy woman with a large (>1 cm), firm supraclavicular lymph node and splenomegaly, most consistent with Hodgkin lymphoma (which is associated with an increased LDH)
  • Key idea: Enlarged supraclavicular lymph node is never normal!
  • Key idea: Hodgkin lymphoma also commonly associated with pruritis (particularly on the NBME exam)

19. A 6-week-old boy is brought to the physician for his first well-child examination…

Fibrosis of the sternomastoid muscle

  • 6 week old newborn who had a difficult delivery via forceps who now has his head rotated to the left with contralateral chin deviation with a nontender mass in the right side of the neck, most consistent with torticollis
  • Torticollis: Neck mass + ipsilateral head tilt + contralateral chin deviation
https://www.correctivechiropractic.com/blog/can-chiropractic-help-torticollis-infants-children/

20. A 3-year-old girl is brought to the emergency department after her father…

Organophosphate

  • Young child who was unsupervised with chemicals and presents with cyanosis, diaphoresis, miosis, rhinorrhea, drooling, signs of bronchoconstriction, and increased bowel sounds, all of which is associated with organophosphate poisoning –> Decreased acetylcholinesterase activity –> Increased acetylcholine –> Increased parasympathetic functions, increased sweating, muscle paralysis (due to repetitive stimulation)
  • Key idea: Treated with atropine (muscarinic receptor antagonist) and pralidoxime (regenerates acetylcholinesterase)
  • Lead poisoning would not present acutely
  • Ethylene glycol –> Anion-gap metabolic acidosis and kidney injury (classically renal stone)
  • Carbon monoxide poisoning –> Cherry-red skin, head, altered mental status

21. A 32-year-old woman, gravida 3, para 1, aborta 1, is brought to the emergency…

Magnesium sulfate

  • Woman with history of preterm labor who presents with preterm labor (contractions AND cervical changes) at 28 weeks’ gestation and therefore should receive (1) Betamethasone (promote lung development) (2) Magnesium sulfate (neuroprotective for baby, tocolytic effects (3) Tocolytics (4) IV Penicillin if GBS positive or unknown
  • Delivery at <32 weeks: See above
  • Delivery at 32-34: (1) Betamethasone (promote lung development) (2) Tocolytics (3) IV penicillin if GBS positive or unknown
  • Delivery at 34-37: (1) Betamethasone (promote lung development) (2) IV penicillin if GBS positive or unknown

22. A male newborn has bilateral clubfoot deformity. He was born at term following…

Spinal dysraphism

  • Male newborn who has lower limb paralysis and numbness with a full bladder, most consistent with a spinal cord injury affecting the lower extremities
  • Spinal dysraphism = Type of neural tube defect
  • Cerebral palsy: Delayed motor milestones, comorbid seizures/intellectual disability, abnormal muscle tone
  • Muscular dystrophy: Waddling gait, hip weakness, Gower sign in a toddler

23. A 42-year-old woman comes to the physician because of a 3-month history of…

Oral propylthiouracil therapy

  • Young woman with subacute palpitations, anxiety, weight loss, heat intolerance, diarrhea, lid lag and exophthalmos with a large, nontender thyroid gland with diffuse increased uptake and low TSH with increased T4 levels, most consistent with Graves’ disease
  • Graves disease and other forms of hyperthyroidism should be treated with propylthiouracil or methimazole
  • Key idea: All causes of hyperthyroidism can lead to lid lag and lid retraction secondary to sympathetic activation of the superior tarsal muscle, but Graves specifically leads to exophthalmos and impaired extra-ocular motion due to glycosaminoglycan deposition behind the eyes
https://www.aafp.org/afp/2016/0301/p363.html

24. A 42-year-old woman is brought to the emergency department because of…

Optic neuritis

  • Young woman with history of neurologic symptoms disseminated in time and space who presents with left eye pain and vision changes who has a central scotoma, an afferent pupillary defect and pallor of the left optic disc most consistent with optic neuritis in the setting of multiple sclerosis
  • Macular degeneration: Older patient with loss of central vision and straight lines appearing curved
  • Retinal detachment: Patient with history of “flashes and floaters” who presents with monocular vision loss like a “curtain drawn down”

25. A 47-year-old man comes to the emergency department because of a 3-day…

Hypovolemia

  • Patient with a 3-day history of profuse nausea/vomiting after significant alcohol consumption who now presents with abdominal pain, hypotension, tachycardia and cool/clammy skin, most consistent with hypovolemic shock
  • Cardiogenic shock: Signs of volume overload (jugular venous distention, lower extremity edema, etc.)
  • Septic shock: Could be seen if patient had sign of perforation (guarding, rebound) and patient would likely have warm and dry skin (because cardiac output is normal/increased and peripheral arterioles are dilated)

26. An otherwise healthy 37-year-old man comes to the physician because of…

Spondylolisthesis

  • Spondylolisthesis: Palpable step-off, pain with spinal extension, with lateral spinal x-ray showing forward slippage of one of the vertebral bodies
  • Key idea: Due to pars intercularis defects that causes forward slip of vertebral body, leading to nerve impingement /// Often seen in adolescent boys
  • Compression fracture: Patient with risk factors for osteoporosis with tenderness upon palpation of the spine and pain with sitting/standing
http://www.learningradiology.com/archives06/COW%20204-Spondolytic%20Spondylolisthesis/spondylocorrect.htm

27. A previously healthy 21-year-old woman comes to the physician 1 day…

Viral pleurisy

  • Previously healthy young patient presenting with severe pleuritic chest pain, shortness-of-breath, mild fever and a faint erythematous rash, consistent with a viral syndrome leading to viral pleurisy and a viral exanthem
  • Key idea: Pleural friction rub = Pleural inflammation = Pleurisy (similar to how pericardial friction rub = pericarditis)

28. A 17-year-old boy comes to the physician because of a 4-day history of severe…

Pill-induced esophagitis

  • Patient who has been taking ibuprofen (NSAIDs) who presents with painful swallowing with no other findings, most concerning for pill-induced esophagitis
  • Causes of pill-induced esophagitis: Tetracyclines, Bisphosphonates, potassium chloride, NSAIDs, Iron
  • Zenker diverticulum: Older patient with halitosis, regurgitation of food +/- palpable neck mass

29. A 27-year-old nulligravid woman comes for a routine health maintenance…

Colposcopy

  • Negative pap smear: Repeat screen in 3 years (or 5 years with HPV co-testing in women 30-65 years old)
  • Atypical squamous cells of undetermined significance (ASCUS): Either perform HPV testing or repeat pap within 6 months
  • Abnormal pap smear (LSIL or worse): Confirmatory colposcopy –> Cryotherapy or loop electrosurgical excision procedure (LEEP) if patient has local ectocervical disease vs. Cone biopsy if patient has local endocervical disease

30. A 27-year-old man is brought to the emergency department by parademics 30…

Transfusion of group O, Rh-negative packed red blood cells

  • Key idea: Patient has active bleed into chest (right hemopneumothorax with negative FAST) that is not responding to fluids (remains tachycardic and hypotensive after 3L crystalloid), and therefore should be managed with blood product resuscitation + surgical intervention
  • Key idea: Group O, Rh-negative blood can be given to patients of all ABO and Rh blood types

31. A 77-year-old man is brought to the emergency department 15 minutes after…

Destruction and dilation of distal airways

  • Older patient with a lifelong history of recurrent episodes of pneumonia and chronic productive cough (which is consistent with bronchiectasis) who presents with self-limited large-volume hemoptysis and is found to have thin-walled cystic spaces in the RLL with air-fluid levels on chest x-ray, most consistent with bronchiectasis due to airway destruction/dilation secondary to repeated pneumonia
  • Key idea: Chronic productive cough = Bronchiectasis (recurrent infections, cystic fibrosis, etc.) or chronic bronchitis (significant smoking history)
https://en.wikipedia.org/wiki/Bronchiectasis

32. A 10-year-old girl is admitted to the hospital because of fever and joint pain…

Mitral valve incompetence

  • Child with strep throat (throat culture and increased ASO titer) who presents with joint pain and a new 3/6 holosystolic murmur at the midclavicular line, most consistent with mitral valve regurgitation caused by rheumatic heart disease
  • Rheumatic heart disease: JONES criteria (Joint pain, O for heart (endocarditis, myocarditis, pericarditis), subcutaneous Nodules, Erythema marginatum, and Sydenham chorea
  • Systolic murmurs (left-sided): Aortic stenosis vs mitral regurgitation

33. A 4-year-old boy develops chickenpox 8 hours after visiting her newborn…

No intervention is necessary

  • Key idea: All mothers have previous exposure to chickenpox and should theoretically have protective IgG that they can pass to their newborns
  • Key idea: Only time newborn should receive prophylactic VZV immunoglobulin is if mother developed chickenpox rash <5 days before delivery, mother developed chickenpox rash <2 days after delivery or if baby were born to a non-immune mother and was exposed to somebody with chickenpox

34. An 87-year-old man comes to the physician because of a 1-year history of…

Finasteride

  • Older man with nocturia, hesitancy, and uncontrollable leaking after the end of urination who is found to have an enlarged prostate on exam, most consistent with benign prostatic hyperplasia
  • 2 potential treatments are (1) 5-alpha reductase inhibitors (finasteride, dutasteride): Lead to reduced conversion of testosterone –> DHT –> Less BPH over the course of months (2) Alpha-1 selective inhibitors (-zosin and tamsulosin): Relaxation of the bladder internal sphincter –> less urethral resistance when voiding
  • Key idea: -zosin drugs (NOT TAMSULOSIN) also have blood pressure effects (because alpha-1 receptors lead to arteriolar constriction) and commonly lead to orthostatic hypotension and dizziness, so in this patient with orthostatic hypotension due to autonomic dysfunction, we would opt for tamsulosin or a 5-alpha reductase inhibitor

35. A 67-year-old man has had an ulcer on the anterior surface of the leg just above…

Stasis dermatitis with ulcer

  • Elderly patient with previous mitral valve dysfunction and history of CHF who presents with bilateral lower extremity edema and an ulcer on the lower medial extremity, most consistent with a venous stasis ulcer
  • 3 main types of lower extremity ulcers include (1) Neuropathic ulcers: Seen in diabetic patient, occur on the sole of the foot especially where lots of pressure is placed while walking (such as proximal to big toe) (2) Arterial ulcers: Patient with signs of peripheral vascular disease (claudication, shiny hairless legs) who has ulcer with necrosis often at tips of the toes (3) Venous ulcers: Often seen in patient with chronic lower extremity edema and leads to ulceration on the medial aspect of the leg

36. A study is proposed to assess the effectiveness of a new vaccine for the prevention…

Coercion of a vulnerable population

  • Prisoners, homeless patients, elderly patients, immigrants, etc. are vulnerable populations

37. A previously healthy 67-year-old man has had an aching burning sensation in…

Bronchogenic carcinoma

  • Older patient with clubbing and painful arthropathy of large joints in the lower extremity relieved by elevation, most consistent with hypertrophic pulmonary osteoarthropathy secondary to lung cancer
  • Key idea: Next best step would be chest x-ray to look for lung cancer

38. A 65-year-old man comes to the emergency department 30 minutes after an episode…

Internal carotid artery

  • Elderly man with vasculopathic risk factors (elevated cholesterol, type 2 diabetes) who presents with transient blindness of the left eye (suggestive of amaurosis fugax due to embolic phenomenon ipsilaterally on the left side) and an episode of aphasia (suggestive of embolic phenomenon affecting the left side of the brain where Broca’s area is located) also found to have a Hollenhorst plaque in the left eye, all of which is consistent with carotid artery stenosis of the left internal carotid artery throwing off small plaques
  • Key idea: Internal carotid artery will become the anterior cerebral artery and the middle cerebral artery
  • Key idea: Amaurosis fugax is highly associated with carotid artery stenosis and is due to occlusion of the ophthalmic artery, which is the first branch off the INTERNAL carotid artery

39. A 62-year-old woman is brought to the emergency department because of a 4-day…

Radiation therapy

  • Older woman with known history of metastatic breast cancer (lymph node positive) who presents with progressive back pain with lower extremity UMN weakness (positive Babinski, brink reflexes) and lower extremity numbness/tingling, concerning for metastases to the vertebral bodies leading to spinal cord compression
  • Key idea: Bone metastatic disease should be managed with radiation therapy
  • https://www.ncbi.nlm.nih.gov/pmc/articles/PMC5421962/

40. A 51-year-old woman had a 15-minute episode of acute right-sided chest pain…

Observation

  • Middle-aged woman who had a right subclavian catheter placed (which is common NBME risk factor for pneumothorax) and then developed right-sided chest pain and dyspnea with normal O2 saturation and a small (10%) apical pneumothorax who should be managed with observation
  • Key idea: If patient had a tension pneumothorax (medical emergency requiring immediate needle decompression or chest tube placement), they would have hemodynamic instability and tracheal deviation away from the affected side
  • Key idea: Patients with a small, largely asymptomatic pneumothorax can be conservatively managed with observation and repeat chest x-ray hours later
  • https://www.mrinz.ac.nz/2020/01/30/psp-nejm/

41. A 62-year-old man comes to the physician because of a 12-hour history…

Vancomycin

  • Patient with immunosuppression (chemotherapy) and repeated insertion of a catheter (notorious for causing nosocomial Staph and Strep infections) who presents with fever and discharge from catheter site found to have an infection with gram-positive cocci in clusters (= Staph aureus or Staph epidermidis) who should be initially treated with Vancomycin to cover MRSA until sensitivities return
  • Key idea: Rates of MRSA are higher in patients who develop nosocomial Staph infections compared to community Staph infections
  • The only cephalosporin that covers MRSA is ceftaroline (5th generation cephalosporin)

42. A 25-year-old woman comes to the physician because of tremulousness and fatigue…

Surreptitious administration of thyroxine

  • These patients have a small thyroid because chronically low TSH leads to low levels of thyroid stimulation and have low iodine uptake into the thyroid because the thyroid is not actively producing thyroid hormone (thyroid hormone is coming into body exogenously)
  • Key idea: We would expect this patient to have low TSH because of negative feedback of exogenous T3/T4 upon anterior pituitary
  • Ophthalmic examination is normal = Not Graves disease
https://www.aafp.org/afp/2016/0301/p363.html

43. A 55-year-old man comes to the physician because of fever, neck pain, and…

Viral infection

  • Patient with a recent viral infection who developed signs of hyperthyroidism (weight loss) and an enlarged, tender thyroid gland, most consistent with DeQuervain thyroiditis
  • Tender thyroid = DeQuervain thyroidits (subacute granulomatous thyroiditis) –> Initially leads to hyperthyroidism due to release of preformed thyroid hormone (low iodine uptake into thyroid gland) followed by hypothyroidism due to inflammation of the thyroid gland

44. A 7-month-old boy is brought to the physician because of a 4-day history of…

Wiskott-Aldrich syndrome

  • Infant with recurring infections, eczema and thrombocytopenia, most consistent with Wiskott-Aldrich syndrome
  • Adenosine deaminase deficiency = Severe combined immunodeficiency –> Failure to thrive, recurrent infections with all types of organisms
  • Ataxia telangiectasia –> Cerebellar defects (ataxia), IgA deficiency (anaphylactic blood transfusion reactions, sinopulmonary infections), spider angiomas
  • DiGeorge syndrome –> CATCH-22 (Cardiac defects, Abnormal faces, Thymic aplasia, Cleft lip/palate, Hypocalcemia (due to lack of parathyroid glands)
  • Hyper-IgE –> Cold staph abscesses (contrast with abscesses without pus seen in leukocyte adhesion defect), coarse faces, retained primary teeth, easy bone fractures
  • Infant HIV –> Chronic diarrhea + failure to thrive + opportunistic infections (thrush, etc.)
  • X-linked agammaglobulinemia –> Patient with history of uncles who die of infection who develops recurrent bacterial (due to lack of IgG) and mucosal (due to lack of IgA) infections found to have decreased levels of all immunoglobulins

45. A 72-year-old woman has hyponatremia 3 days after admission to the hospital…

Inappropriate ADH (vasopressin) secretion

  • Elderly patient with cerebral disease who has hypo-osmolar hyponatremic with increased urine sodium (>40) and urine osmolarity >100, consistent with SIADH
  • Key idea: Causes of SIADH include ectopic ADH (small cell lung cancer), pulmonary disease, CNS disorders or head trauma (such as infarction), and drugs (cyclophosphamide)
  • Dehydration –> Low urine sodium (<40) because the body is trying to retain as much sodium and water as possible
  • Water intoxication –> Low urine osmolality (often <100) because so much water is being consumed and excreted that the urine is very dilute

46. A health status survey compares the clinical outcomes of patients treated…

The results are not adjusted for comorbidities

  • Process of elimination: Study period of 1 year does not seem short, results are significant (p < 0.05), study is likely well-powered with sample size >500 and the focus of the study is clinical (physical functioning, mobility)

We are not affiliated with the NBME, USMLE or AAMC.

The answer explanations may not be reproduced or distributed, in whole or in part, without written permission of Step Prep.